1
$\begingroup$

Schauder's lemma asserts that you can always extend a uniformly continuous, uniformly open map from a dense subset of a complete metric space to a uniformly open map on the completion.

I think the converse should be false: is there an example of a uniformly continuous uniformly open map from a complete metric space $f\colon X\to Y$ whose restriction $f|_D\colon D\to f(D)$ to some dense set $D\subseteq X$ fails to be uniformly open?

Edit in regard to ${\rm id}_{\mathbb R}|_{\mathbb Q}$. Suppose that $f\colon X\to Y$ is an injective uniformly open map. Then for each $\varepsilon > 0$ there is $\delta > 0$ such that for all $x$ we have $f(B(x,\varepsilon))\supseteq B(f(x), \delta)$. Let $D$ be a dense subset of $X$. Then for $x\in D$ we have $$f(B_D(x,\varepsilon)) = f(B(x,\varepsilon)\cap D) = f(B(x,\varepsilon))\cap f(D)\supseteq B(f(x), \delta) \cap f(D) = B_{f(D)}(f(x), \delta).$$ Here we used injectivity to make the image and intersection commute.

$\endgroup$
6
  • 2
    $\begingroup$ What about the identity map of the real line and its restriction to the set of all rational numbers? $\endgroup$ Commented Dec 29, 2022 at 19:01
  • $\begingroup$ @IosifPinelis, looks like for injective maps this is not a problem. Or am I missing something? $\endgroup$ Commented Dec 29, 2022 at 23:00
  • 1
    $\begingroup$ For me, the Schauder lemma is the following statement for a continuous map $f:X\to Y$ from a complete metric space $X$ to a metric space $Y$: If $f$ is uniformly almost open in the sense that, for every $\varepsilon>0$, there is $\delta>0$ such that $B_Y(f(x),\delta) \subseteq \overline{f(B_X(x,\varepsilon))}$ for all $x\in X$, then $f$ is uniformly open, i.e., the same condition holds without the closure. $\endgroup$ Commented Dec 30, 2022 at 7:41
  • 2
    $\begingroup$ @IosifPinelis, I think that misunderstanding emerged from the fact you wanted to keep the codomain unchanged, which was not my intention (as otherwise this is indeed trivially false). $\endgroup$ Commented Dec 30, 2022 at 9:06
  • $\begingroup$ @user243245 : According to the standard definition (en.wikipedia.org/wiki/…) of the restriction of a function to a subset of the domain, the codomain is kept -- and why not, given that in general the codomain does not have to coincide with the image of a function? Anyhow, I see that now you have specified your notion of a restriction (this should have been done right away when you posted the question). $\endgroup$ Commented Dec 30, 2022 at 13:31

0

You must log in to answer this question.

Start asking to get answers

Find the answer to your question by asking.

Ask question

Explore related questions

See similar questions with these tags.